Peds (Exam 02)

Pataasin ang iyong marka sa homework at exams ngayon gamit ang Quizwiz!

A woman comes in for her annual gynecological examination and informs the nurse that she is going to try and become pregnant. She asks the nurse when the best time in the month is to become pregnant. What is the best response by the nurse?

"3 days before until 2 days after ovulation."

A parent of a 10 year old tells the nurse, "I don't know what to do about my child's teeth. They always have cavities and problems." What is the best response by the nurse?

"A balanced diet with plenty of calcium and phosphorus and minimal sugar may help."

A client at 39 weeks' gestation calls the OB triage and questions the nurse concerning a bloody mucous discharge noted in the toilet after an OB office visit several hours earlier. What is the best response from the triage nurse?

"A one time discharge of bloody mucus in the toilet might have been your mucous plug."

A woman is to undergo chorionic villus sampling as part of a risk assessment for genetic disorders. What statement would the nurse include when describing this test to the woman?

"A small piece of tissue from the fetal placenta will be removed and analyzed."

The school nurse is conducting a health class with a group of high school students on the topic of sex and sexuality issues. The nurse determines the session is successful when the students correctly choose which aspect as most important?

"Adolescents need to know how to use condoms correctly to avoid sexually transmitted infections."

A pregnant woman undergoing amniocentesis asks her nurse why the baby needs this fluid. What would be an accurate response from the nurse?

"Amniotic fluid cushions your baby to prevent injury."

A 27-year-old client is in the first trimester of an unplanned pregnancy. She acknowledges that it would be best if she were to quit smoking now that she is pregnant, but states that it would be too difficult given her 13 pack-year history and circle of friends who also smoke. She asks the nurse, "Why exactly is it so important for me to quit? I know lots of smokers who have happy, healthy babies." What can the nurse tell the client about the potential effects of smoking in pregnancy?

"Babies of women who smoke tend to weigh significantly less than other infants."

A woman at 15 weeks' gestation who works at a daycare center thinks she may have just been exposed to rubella at work. The client asks how this may affect her fetus. What is the best response the nurse can give?

"By the end of the eighth week all of the organ systems and major structures are present, so exposure to any teratogen can lead to birth defects. More assessments are needed."

The mother of a 6-year-old is asking the nurse how to handle the child's lying and fabricated stories when confronted with questionable actions. Which response would be most appropriate by the nurse?

"Children this age sometimes can't distinguish between fantasy and reality."

The school nurse is presenting a lecture to adolescents to teach them how conception occurs. Which statement by the nurse would accurately describe this process?

"Conception usually occurs when the ovum is in the outer third of the fallopian tube."

A primigravida who is 40 weeks' pregnant thinks she may be in labor. She calls the nurse and reports that she has had 10 contractions in the last hour. She says that they are mildly painful and last 45 seconds. What is the best response from the nurse?

"Did your water break? Do you feel the baby moving?"

A nursing instructor is teaching about causes of infertility and identifies a need for further instruction when a student states which of the following?

"Diet does not play a role in infertility."

The nurse is assessing the gross motor skills of an 8-year-old boy. Which interview question would facilitate this assessment?

"Do you participate in any sports?"

A nursing student is learning about teaching and how to evaluate is effectiveness. Which of the following statements made by the student identifies a need for further instruction

"Evaluation is done as a final step of teaching only."

A nursing student is learning about teaching and how to evaluate is effectiveness. Which of the following statements made by the student identifies a need for further instruction?

"Evaluation is done as a final step of teaching only."

The nurse is presenting information about school-aged children at a community event. Which statement should the nurse prioritize for further teaching and providing more information?

"Food is so expensive, we always make our children eat everything on their plates."

The nurse is caring for an 11-year-old girl. The girl's mother reports that the girl does not want to play team sports like soccer or volleyball anymore. Her daughter insists she does not enjoy them. The mother is concerned that her daughter will not get enough physical activity and asks the nurse for guidance. How should the nurse respond?

"Give her some options; it's important to find something she enjoys."

The mother of a 12-year-old boy is talking with the school nurse about her son's clumsiness. She reports that he seems to fall a lot, his writing is horrible, and as much as he practices he can't play his guitar very well. How should the nurse respond to the mother?

"Have you spoken with your pediatrician about your observations?"

A pregnant client in her first trimester is being seen at the women's clinic. What statement by the nurse would be most beneficial to this client to support her acceptance of the pregnancy?

"Having feelings of uncertainty are very common when a woman becomes pregnant. I will be glad to talk to you about any concerns you may have."

In conducting a health assessment of a 6-year-old girl, the nurse wants to determine if the parents have any concerns about her development. How can the nurse best elicit their concerns?

"How do you feel about your daughter's overall development?"

The nurse is providing education about barrier contraceptives. Which statement by the patient indicates a need for further education?

"I can leave my diaphragm in for up to 48 hours."

A female client who has chosen the cervical cap as her method of contraception states that she knows how to use it. Which statement by the client would indicate to the nurse that the client does understand this method of contraception?

"I do not have any spermicidal allergies, so this method is OK for me."

The school nurse is the guest speaker in a classroom of fourth-grade children. They are discussing eating habits and the number of servings from each of the food groups that is appropriate for the school-age child. Which statement made by the children best indicates an appropriate intake of one of the food groups for this age child?

"I drink three big glasses of milk each day."

Monica is 36 weeks pregnant. Which complaint by Monica requires immediate additional assessment by the nurse midwife?

"I have been leaking clear, vaginal fluid."

A pregnant woman at her first prenatal visit asks the nurse if it is safe to have sex during her pregnancy. Which client statement alerts the nurse to the need for further teaching?

"I should substitute intercourse with nonsexual touch to avoid harming the fetus."

The nurse is teaching a group of school age children about physical development. Which statement made by one of the children indicates the correct understanding of the teaching?

"I will grow an average of 2 inches per year"

After teaching a woman who has chosen the vaginal ring as her method of contraception, the nurse determines that the client needs additional teaching when she makes which statement?

"I will insert a new ring at the same time and day of every week."

The nurse is teaching the pregnant woman about nutrition for herself and her baby. Which statement by the woman indicates that the teaching was effective?

"I will need to take iron supplementation throughout my pregnancy even if I am not anemic."

A 14-year-old boy is far quieter than usual when he visits his physician's office. The nurse asks him if anything is wrong, and the boy replies that he did not make the cut for the football team. What would be most appropriate for the nurse to say?

"I'm really sorry; I know that must be a big disappointment."

After assessing a woman who has come to the clinic, the nurse suspects that the woman is experiencing abnormal uterine bleeding. Which statement by the client would support the nurse's suspicions?

"I've been having bleeding off and on that's irregular and sometimes heavy."

A nursing instructor is teaching about fertility and realizes a need for further instruction when a student states:

"In most couples with a subfertility problem, it is the man who is subfertile."

Nursing students are learning about the importance of therapeutic communication in their pediatric course. The nursing instructor identifies a need for further teaching when a student makes which statement?

"It is best to stand when listening to a child to demonstrate knowledge."

The nurse is promoting nutrition to a 13-year-old boy who is overweight. Which instruction should the nurse expect to include in the discussion?

"Keep a food diary."

The nurse is caring for an 8-year-old girl. She is reviewing her nutritional requirements and describing interventions that promote healthy eating habits. Which response by the girl's mother indicates a need for further discussion?

"My daughter must stay at the table until she has cleaned her plate."

The nurse is caring for a 6-year-old boy. During the course of a routine wellness examination, the mother proudly reports that the child eats whatever the mother puts on his plate. The nurse wants to emphasize the importance of allowing the child to make some of his own choices regarding the types of foods he eats. How should the nurse communicate this to the mother?

"Now is the time to let him choose some of his meals."

A couple is seeking guidance for their inability to conceive a child after trying for 15 months. They are morbidly obese but state they have friends with the same weight problem who have had no difficulty conceiving. What education can the nurse provide this couple to increase their chances of success?

"Obesity may interfere with effective penetration and deposition of sperm. We will look at several factors to discover what issues you may be encountering."

During a visit to the nurse practitioner for a sports physical, the father of a 9-year-old boy tells the nurse that there is no need for his son to engage in physical activity because he plays soccer in the local soccer league. What is the best response from the nurse?

"Organized sports should serve as complementary activities and not replace activity."

The nurse is teaching a couple about the pros and cons of genetic testing. Which statement by the nurse best describes the limits of genetic testing?

"Some genetic tests can give a probability for developing a disorder."

The nurse has provided information to a client about oral contraceptives. Which statement by the client would indicate a need for further education?

"Some oral contraceptives protect against STIs."

During a health check-up without his parents present, a 17-year-old tells the nurse he is gay. Which initial question is best?

"Tell me what makes you think you are gay."

The nutritionist is presenting information about vegetarian diets with a group of nursing students. The nutritionist determines the session is successful when the students correctly choose which factor concerning the semi-vegetarian diet?

"The diet excludes red meat and possibly poultry."

A client who is 32 weeks gestation tells the nurse that she has been experiencing shortness of breath when walking up the steps at home. She is concerned that something is wrong. What is the nurse's best response?

"The enlarging uterus pushes against your diaphragm and this makes breathing shallow"

The nurse is teaching a prenatal class about preparing for their expanding families. What is helpful advice from the nurse?

"The hormones of pregnancy may cause anxiety or depression postpartum."

A female client comes to the clinic and asks the nurse what would be the most cost effective method for her to determine her ovulation patterns. What is the best response by the nurse?

"The least costly method to determine ovulation patterns is to record the basal body temperature for at least 4 months."

A young client in the infertility clinic tells the nurse, "I just know it is my fault we cannot conceive." What would be the best response by the nurse?

"The problem can be with the man, woman, or both."

During an annual visit of a 6-year-old boy, the nurse observes dental caries on two of the child's primary teeth. Which response by the parents suggests more education is needed regarding the importance of primary teeth?

"These are only his baby teeth so we are not worried."

Which statement is most appropriate when initiating a nursing action with a preschooler?

"These sticky snaps are for your chest."

During the annual physical examination, a teenage client tells the nurse that she is anxious as well as frustrated because of the acne on her face. Which is the best response for the nurse?

"This is one of the most common physical changes during adolescence."

A mother brings her 8-year-old daughter into the doctor's office because over the past year her tonsils have increased in size to the point that the mother is concerned that her breathing will be obstructed. The girl has no pain, fever or other symptoms. Following this data collection, which instruction is best?

"This may be normal growth of lymphatic tissue for this age."

The pregnant client at 6 weeks' gestation asks the nurse if an ultrasound will reveal the sex of the fetus yet. What is the best response by the nurse?

"We will have to wait until the baby is 16 weeks' gestation to determine what the sex is."

Which statement by the nurse encourages therapeutic communication from a child scheduled for surgery?

"What are you worried about?"

During a health maintenance visit, a 15-year-old girl mentions that she is not happy with being overweight. Which approach is best for the nurse to take?

"What specifically have you been noticing?"

Amanda is about 16 weeks pregnant and is concerned because she feels her "abdomen" contracting. She calls the primary care provider's office and speaks to the nurse. What is the nurse's most appropriate response to Amanda's concern?

"What you are feeling are called Braxton Hicks contractions. They are considered practice contractions during pregnancy."

A pregnant woman in her first trimester comes to the clinic for a visit. Which statement would lead the nurse to suspect that the woman is experiencing a complication of pregnancy that requires additional follow up?

"When I first urinate in the morning, I notice a burning sensation, but then as I'm constantly going throughout the day, it goes away."

The school nurse is meeting with a group of 11-year-old girls to discuss expected puberty changes in their bodies. When one of the girls states, "I just feel like my whole body is changing and I don't know why" what should the nurse point out to this group?

"You have lots of hormone changes going on right now."

The mother of a 6-year-old girl tells the nurse that she is very concerned that her daughter develops good self-esteem. Which nursing instruction is best?

"You need to provide guidance and celebrate successes."

A patient is 12 weeks' pregnant and asks the nurse about dental care. Which is the best response?

"You should continue to see your dentist regularly for preventive care, but avoid any nonemergent dental work until after the baby is born."

A client in her first trimester of pregnancy visits the healthcare facility. The client informs the nurse that she has a pet cat at home and wants to know if it poses any threat to the fetus. Which response by the nurse would be most appropriate?

"You should not empty the cat's litter box."

A client in her second trimester of pregnancy arrives at the health care facility for a routine follow-up visit. The nurse is required to educate the client so that the client knows what to expect during her second trimester. Which information should the nurse offer?

"You will experience quickening, and you will actually feel the baby."

The nurse is enlisting the parents' assistance for therapeutic hugging prior to an otoscopic examination. What should the nurse emphasize to the parents?

"You will need to keep his hands down and his head still."

The nurse is preparing a client for a chorionic villi sampling procedure. Which factor should the nurse point out in the teaching session to the client?

"You'll have an ultrasound first and then the test."

The nurse is assessing a 16-year-old girl at an annual well-clinic visit and notes the girl started menses at 13 years of age and grew 1 inch (2.5 cm) over the past year. When questioned by the young lady if this is normal, which answer should the nurse prioritize?

"You're following expected patterns of growth."

A nurse is teaching a client who has had a Copper IUD inserted. The nurse will teach the client that the device can stay in place and be effective for up to how long? Fill in the blank with a number.

10

A nurse is reviewing sperm count results for a couple trying to conceive. Which results would concern the nurse?

10 million per mL of seminal fluid

A pregnant client is scheduled to undergo chorionic villi sampling (CVS) to rule out any birth defects. Ideally, when should this testing be completed?

10 to 12 weeks of gestation

According to the Family and Medical Leave Act, a woman is guaranteed the right to how many weeks of unpaid, job-protected leave for the birth of a child?

12

Subfertility/infertility is said to exist when a couple has failed to achieve pregnancy after how many months of unprotected sexual intercourse?

12

A nursing instructor is teaching about subfertility to nursing students and perceives their understanding when they define subfertility existing when a pregnancy has not occurred after how much time of engaging in unprotected coitus?

12 months

The nurse is teaching a pregnant woman with a prepregnancy body mass index (BMI) of 26 about recommended weight gain. The nurse determines that the teaching was successful when the woman states that she should gain approximately how much during her pregnancy?

15 to 25 pounds (7 to 11 kilograms)

A male client has undergone a semen analysis for evaluation of fertility. The nurse understands that which sperm count would suggest infertility?

18 million/mL

A nursing student learning pediatrics and the development of language correctly identifies the age when children are able to put together two-word (noun-verb) sentences to be:

2 years

A urinalysis is done on a client in her third trimester. Which result would be considered abnormal?

2+ Protein in urine

A nursing instructor is discussing weight gain during pregnancy with a group of college students in a health class. The instructor determines the session is successful when the students correctly point out a woman with a BMI of 21 should have the goal of gaining how much additional weight during the pregnancy?

25 to 35 pounds (11 to 18 kilograms)

"Conception usually occurs when the ovum is in the outer third of the fallopian tube."

4 weeks ago

A nurse is assessing a man who, along with his partner, has been having trouble conceiving. When assessing the man, which area would the nurse most likely include as possibly affecting his fertility?

A 24-hour food intake History of a sexually transmitted disease Testes exposed to radiation A job that requires sitting all day

The nurse has assessed several clients who have arrived for routine appointments. The nurse predicts the health care provider will prioritize a bone density scan for which client?

A 55-year-old Caucasian smoker, family history of osteoporosis

A hospitalized 7-year-old is recovering from a head injury. Occupational therapy has been ordered to assist the child in regaining eye/hand coordination. If the child cannot master this skill, what feelings may arise?

A feeling of inferiority

A female client is having a procedure this morning that involves radiologic examination of the fallopian tubes using a radiopaque medium. What procedure should the nurse document as being performed?

A hysterosalpingography

A nurse is caring for a hospitalized 7-year-old whose family members have been unable to visit for 2 days. The nurse is preparing a diversional activity for the child. Which activity would best be suited for a child in this age group?

A paint-by-numbers activity creating a picture

Which teaching strategy would be most effective in teaching preschoolers about hospitalization?

A puppet show

A nurse is preparing a client for intrauterine device (IUD) insertion. What should the nurse inform the client when educating her on IUDs?

A regular check of threads must be done.

The nurse at an elementary school is explaining the concept of industry versus inferiority to a group of nursing students. What is part of this stage of Erikson's theory?

A sense of competence, mastery, and worth

Hormone levels of a woman indicate that the corpus luteum stopped functioning and releasing progesterone after 5 weeks. The nurse would recognize that which scenario is the expected outcome?

A spontaneous abortion (miscarriage) would occur.

A nurse is counseling women on birth control choices. Which woman is the best candidate for an IUD?

A woman who has one partner and three children

A client prescribed COC has presented for a routine visit. Which finding upon assessment should the nurse prioritize?

Abdominal pain

The school nurse is providing school health screenings to 7- to 11-year-olds exhibiting behaviors in Piaget's stage of concrete operational thoughts. What behaviors are anticipated in normal development? Select all that apply.

Ability to assimilate and coordinate information about the world from different dimensions Ability to see things from another person's point of view and think through an action Ability to use stored memories of past experiences to evaluate and interpret present situations Ability to understand the principle of conservation—that matter does not change when its form changes

What finding would the nurse most likely discover in a 10-year-old child in the period of concrete operational thought?

Ability to classify similar objects

What medication can the nurse tell the patient that, when taken as directed, is non-teratogenic and safe during pregnancy?

Acetaminophen

A nurse is explaining cognitive development in children to a client, with the help of Piaget's theory of cognitive development. What would be the best explanation by the nurse about the formal operations level of cognitive development?

After age 12 children can think in the abstract, including complex problem solving.

The use of what during pregnancy is potentially most harmful to the fetus?

Alcohol

Place the following events in the sequence the pregnant woman would experience them, from first to last. All options must be used.

Amennorhea Uterine enlargement Quickening Braxton Hicks contractions Labor

The fluid-filled, inner membrane sac surrounding the fetus is which structure?

Amnion

A pregnant patient is undergoing a fetal biophysical profile. Which parameter of the profile helps measure long-term adequacy of the placental function?

Amniotic fluid volume

The nursing instructor is leading a discussion on the process of implantation with a group of nursing students. The instructor determines the session is successful after the students correctly choose which explanation of a blastocyst?

An inner layer of cells that is separated from an outer layer of cells by a fluid-filled cavity

A woman who has just found out that she is pregnant tells the nurse that she takes docusate sodium (colace). The nurse identifies this drug as a category C medication. Which information best reflects the nurse's understanding of this category?

Animal studies have shown an adverse effect on the fetus, but no adequate studies have been done in humans. Pregnancy risk is unknown.

A nurse is conducting a class for a group of couples about subfertility. When describing the causes of subfertility, which cause would the nurse include as being most common?

Anovulation

A young woman is having trouble conceiving and has come in to the office for fertility testing. She states that she has irregular and unpredictable menstrual cycles. On blood testing, it is found that her ovaries are producing excess testosterone, which is lowering her follicle-stimulating hormone (FSH) and luteinizing hormone (LH) levels. Which of the following is the most likely cause of her subfertility?

Anovulation

The nurse is admitting a 10-year-old for surgery. What action should the nurse prioritize when caring for this child?

Answer questions regarding pain.

A teratogen is _____. Choose the BEST answer.

Any chemical or physical factor that can potentially adversely affect the developing fetus

A child who is scheduled for an x-ray repeatedly talks about how she is not worried about the procedure. What is the priority action by the nurse?

Ask the child if she is concerned or worried, or has any questions about the x-ray

A client is scheduled for amniocentesis. Which action by the nurse would be most appropriate when preparing the client for the procedure? Select all that apply:

Ask the client to void. Assess fetal heart rate.

During a routine antepartal visit, a pregnant woman reports a white, thick vaginal discharge. What would the nurse do next?

Ask the woman if she is having any itching or irritation.

A woman has just learned that she is pregnant and would like to know how soon she can find out via ultrasound the sex of her fetus. The nurse should respond with which of the following?

At about 4 months

The nursing instructor is presenting a session on the cellular division involved in the reproduction of human life. Which statement indicates the group's need for further education?

At ovulation, the gametes unite to form the cell that becomes the developing fetus.

A woman is at 20 weeks' gestation. The nurse would expect to find the fundus at which area?

At the level of the umbilicus

What instruction should a nurse offer to a pregnant client or a client who wishes to become pregnant to help her avoid exposure to teratogenic substances?

Avoid medications.

A nurse is educating a primigravida client about the expected changes during pregnancy. Which measure will provide anticipatory guidance about pregnancy?

Avoid wearing high heels, especially during late pregnancy.

A woman in her third trimester complains to the nurse of significant back pain. The nurse questions the client carefully and records a detailed account of her back symptoms. What is the best rationale for the nurse evaluating the client's back symptoms with such care?

Back pain could be a sign of bladder or kidney infection

The school nurse is preparing a health education session on the topic of birth control for a college group. Which method should the nurse emphasize for this group?

Barrier

The nurse is teaching a young couple, who desire to start their family, the various methods for determining fertility. After discovering the woman regularly travels internationally for work, deals with a lot of job anxiety and frequently uses an electric blanket at home, the nurse will discourage the use of which method?

Basal body temperature method

A nurse is demonstrating dressing changes for a 12 year old so that the child will be able to perform the skill when discharged. What is important for the nurse to do prior to demonstration?

Be sure that all necessary equipment is present to demonstrate the technique.

A nurse is teaching a group of primigravida woman who are in their first trimester. One of the women asks the nurse about sexual activity during pregnancy. Which information would the nurse most likely incorporate into the response?

Because of pelvic congestion, women may experience increased clitoral sensitivity.

The experienced nurse working in a fertility clinic understands the stress couples experience when they have to "schedule" sexual relations. Which suggestion would be most helpful to a couple to help prevent further stress on their relationship?

Begin a new activity together.

The nurse is meeting with a group of caregivers of adolescents and discussing sex and sexuality, including how to discuss these issues with their children. Which comment should the nurse prioritize with this group of caregivers?

Being honest and straightforward with teenagers will encourage them to ask about subjects like sexuality.

A woman at the infertility clinic for the first time asks, "What could have caused my infertility?" Learning has taken place when the woman can identify which common causes of infertility?

Blocked fallopian tubes Ovarian dysfunction

A pregnant client in her third trimester, lying supine on the examination table, suddently grows very short of breath and dizzy. Concerned, she asks the nurse what is happening. Which response should the nurse prioritize?

Blood is trapped in the vena cava in a supine position.

Which change related to the vital signs is expected in pregnant women?

Blood pressure decreases.

The nurse is preparing a presentation for a local health fair depicting the differences in maturity between preadolescents. Which differing factor should the nurse prioritize in the presentation?

Boys grow at a slower, steadier rate than do girls.

What should a patient expect at her first prenatal visit? Choose the best response.

Breast examination, speculum examination, bimanual examination of the uterus, and blood work

Which information provided by a client would be considered a presumptive sign of pregnancy?

Breast tenderness

During a prenatal visit, the nurse inspects the skin of the client's abdomen. Which would the nurse identify as an abnormal finding?

Bruising

All of the following are premonitory signs of labor EXCEPT

Chadwick's sign

Increased pigmentation on the face of some pregnant women is called:

Chloasma

Many changes occur in the body of a pregnant woman. Some of these are changes in the integumentary system. What is one change in the integumentary system called?

Chloasma

A client asks the nurse, "What would be the best way to get pregnant." The nurse would incorporate which of the following in the response?

Conception is more likely when intercourse occurs within 72 hours of ovulation

A nursing instructor informs students that when teaching children, they need to establish expected outcomes that are which of the following? (Select all that apply.)

Concrete Specific Measurable

Which method of contraception is considered a barrier method?

Condom

The nurse is teaching the parents of a 9-year-old girl about the socialization that is occurring in their child through school contacts. Which information would the nurse include in her teaching plan?

Continuous peer relationships provide the most important social interaction for school-age children.

A newly married couple is meeting with the nurse to discuss a temporary method of birth control that is both a natural form and does not employ birth control pills/devices, in keeping with their religious beliefs. Which fertility awareness method should the nurse point out will best meet their request to delay conception until they are ready?

CycleBeads

When describing the various changes that occur in organ systems during adolescence, what would the nurse include?

Decrease in heart rate

A nurse is discussing a 9-year-old girl's obesity with her parents. Which nursing instructions are helpful? Select all that apply.

Decrease intake of approximately 1,200 calories a day Limit time spent on sedentary activity Participate in a formal weight-control program Encourage fun exercise classes

A community health nurse is preparing a presentation for a health fair on the topics of planning for a pregnancy. Which major goal has the nurse determined should be accomplished with this presentation?

Decrease the number of unwanted pregnancies.

When preparing a class for a group of pregnant women about nicotine use during pregnancy, the nurse describes the major risks associated with nicotine use including:

Decreased birth weight in neonates.

A sexually active client comes to the clinic requesting some form of birth control but is concerned about the bad side effects she has heard that birth control pills can cause. Which information should the nurse point out about the positive aspects of this type of contraceptive? Select all that apply.

Decreased incidence of dysmenorrhea Decreased acne Improved cycle regularity Decreased acute pelvic inflammatory disease

Which assessment finding in the pregnant woman at 12 weeks' gestation should the nurse find most concerning? The inability to:

Detect fetal heart sounds with a Doppler.

The nursing instructor is presenting a class on the various classifications of twins. The instructor determines the session is successful after the students correctly choose which classification that indicates twins have separate amniotic sacs and placentas?

Diamniotic-dichorionic

The nurse discusses various contraceptive methods with a client and her partner. Which method would the nurse explain as being available only by prescription?

Diaphragm

The nurse is meeting with a group of adolescent athletes to discuss their nutritional needs. The nurse should encourage the adolescents to include which foods in their diet to increase iron intake? Select all that apply.

Dried fruits Peanuts Hard-boiled eggs

The nurse is teaching a pregnant client in her last trimester about interventions to help reduce risks of complications during pregnancy. Which of the following are recommended guidelines?

During the last month, rest on the left side for at least an hour, morning and afternoon.

A 38-year-old client presents to the clinic desiring to get pregnant. She reports she had a tubal ligation in her early 20s after two babies and a divorce. After learning that the client recently underwent a reversal of the tubal ligation, the nurse will warn the client of which potential risk?

Ectopic pregnancy

A 40-year-old female client with a chronic pelvic infection expresses her desire to conceive post-treatment. When discussing this with the client the nurse keeps in mind that the client is at increased risk for which of the following?

Ectopic pregnancy

What advice can the nurse give a female patient trying to conceive to facilitate sperm collection to the cervix?

Elevate the hips on a small pillow after coitus.

The nursing student is preparing a presentation which will illustrate the various stages of fetal development. The student will label which stage as the time when the various tissues of the growing embryo begin to assume specific functions?

Embryonic

The nurse is assessing a couple who has come to the health care facility because they have been unable to conceive a child. When assessing the woman, the nurse would identify which factor as increasing the woman's risk for infertility?

Endometriosis

A woman at the infertility clinic for the first time asks, "What could have caused my infertility?" After teaching the woman about possible causes, the nurse determines that the teaching was successful when the woman identifies which condition as a common cause?

Endometriosis Pelvic inflammatory disease Anovulation

What physiological changes would be noted in a pregnant woman? Select all that apply.

Enlarged pituitary gland Lordosis Delayed gastric emptying and decreased peristalsis

An 8-year-old boy's foster mother is concerned about three recent cavities found in his permanent teeth and reports the child eats a nutritional diet, doesn't eat junk food, and the town water supply is fluoridated. Which suggestion should the nurse prioritize to this mother in regard to the child's dental health

Ensure that the child brushes his teeth after each meal and snacks.

A physically fit 30-year-old woman in her first trimester would like to continue exercising during pregnancy. She says she normally jogs, but has been thinking about taking up cycling. She also would like to know how much exercise she should get. Which instructions should the nurse give to the client? Select all that apply.

Exercise three times weekly for 30 consecutive minutes. Include warm-up and cool-down exercises. Avoid sports that involve body contact. Avoid taking up a new sport, such as cycling, during pregnancy. Try walking which is an excellent exercise option during pregnancy.

Children can become physically addicted to glue

False

When providing preconception care to a client, the nurse would identify which medication as being safe to continue during pregnancy?

Famotidine

The nurse is counseling a young woman who has just entered her second trimester, after an uneventful first trimester. She tells the nurse, "It still doesn't seem real. It's just hard to believe that I will really have a baby." Which future events should the nurse point out that will help the young woman come to believe it is real? Select all that apply.

Feeling the baby kick Seeing an ultrasound image of the baby

The nurse is developing a presentation for a community group of young adults discussing fetal development and pregnancy. The nurse would identify that the sex of offspring is determined at the time of:

Fertilization

After the nurse describes fetal circulation to a pregnant woman, the woman asks why her fetus has a different circulation pattern than hers. When responding to the client, the nurse integrates understanding of what information?

Fetal circulation carries highly oxygenated blood to vital areas first.

The maternal serum alpha fetoprotein blood test is performed on pregnant women to screen for which condition?

Fetal neural tube defects

Based on Erikson's developmental theory, what is the major developmental task of the adolescent?

Finding an identity

Why does the nurse complete a process recording?

For self-evaluation of therapeutic communication

Fetal circulation differs from the circulatory path of the newborn infant. In utero the fetus has a hole connecting the right and left atria of the heart. This allows oxygenated blood to quickly pass to the major organs of the body. What is this hole called?

Foramen ovale

A 17-year-old boy chats excitedly with the nurse about his plans for college and a career. He says he has checked out every college in the region and determined which one is the best fit for him and would give him the best career options. The nurse recognizes which developmental aspect in this young man?

Formal operational thought

A nurse is caring for a 10 year old with asthma. The child states, "I hate school because I am always sick. Nobody will ever choose me to be on their team." What level of communication is the child displaying?

Fourth Level: Shared Feelings

A nursing instructor is explaining the stages of fetal development to a group of nursing students. The instructor determines the session is successful after the students correctly choose which time period as representing the pre-embyonic stage?

From fertilization and to the end of the second week.

A 7-year-old with sickle-cell disease who comes to the hospital frequently appears withdrawn and depressed. He refuses to talk to anyone or even admit that he is sad. What would be the best thing for the nurse to do that might help the child deal with his feelings?

Get him to draw a picture.

A mother suspects that her 11-year-old son is experimenting with deliriants with his friends. Which symptoms would the nurse advise the mother to look for that would validate her concerns?

Giddiness and coughing

During a vaginal exam, the nurse notes that the lower uterine segment is softened. The nurse documents this finding as:

Hagar sign.

A client at 16 weeks' gestation comes to the office for a routine exam. At what location within the abdomen would the nurse anticipate the uterus to be found?

Halfway between the symphysis pubis and the umbilicus

A 15-year-old boy shows a pattern of gaining weight, not a large amount but a little more each visit. He is not active in any sports and says that he and his father eat out a lot. What is the best way for the nurse to assess the child's eating pattern?

Have the child keep a food diary for one week.

A group of nursing students are reviewing information about bullying and children who are bullied. The students demonstrate a need for additional review when they identify which of the following as characteristic?

Having numerous friends

A woman comes to the prenatal clinic and undergoes a pelvic exam. The doctor notes a softening of the uterine isthmus. The nurse recognized that this finding is known as what sign?

Hegar sign

The nurse is assessing the laboratory test results of a client with abnormal uterine bleeding (AUB). Which finding should the nurse prioritize?

Hemoglobin level of 10.1 g/dL

A nurse assesses a 32-year-old primigravida client with twin gestation in her second trimester. The client reports constipation from iron supplements. Which condition should the nurse assess for in this client as a result of the constipation?

Hemorrhoids

The nurse notes a school-aged child in the pediatric unit playroom is busy sorting through the Legos and putting them in certain groupings. The nurse determines this child is illustrating which developmental concept?

Hierarchical arrangement

A young woman comes to the free clinic asking for birth control pills. Which factor best indicates that another type of contraception would be better for this client?

History of noncompliance with medications

The nurse is assessing a man concerned about his ability to father children because he has an "abnormality of the penis." The nurse finds that the urethral opening is on the ventral surface of the penis. How would the nurse document this finding?

Hypospadias

A clinic nurse explains to a client who is undergoing an infertility workup that the patency of her fallopian tubes will be checked. Which test is currently used to do this?

Hysterosalpingography

A 13-year-old girl is struggling to accept the new changes that are occurring in her body as a result of puberty. After talking with this client, the nurse recalls that Erikson's primary developmental task of early and mid-adolescence is:

Identity versus role confusion.

A nurse is teaching the parent of a 6-year-old with decay in several deciduous teeth about tooth care and the importance of seeing a dentist. What instruction is best for the nurse to inform the parent about the new 6-year molars?

If the 6-year molars become decayed and have to be pulled, the child could have dental problems later.

The school nurse is developing a school wellness program to promote healthy eating habits and regular physical activity. Each class will be engaging in lessons, contests, and goal setting to develop healthy habits. What is the most important element to emphasize to maximize compliance and long-term change?

Include both parents and children in the wellness program.

During a routine visit to the clinic, a client tells the nurse that she thinks she may be pregnant. The physician prescribes a pregnancy test. The nurse should know the purpose of this test is to determine which change in the client's hormone level?

Increase in human chorionic gonadotropin (hCG)

Which physical change would the nurse expect to find in a pregnant client? Select all that apply.

Increased blood volume Supine hypotension

The nurse has seen a 15-year-old girl and a 16-year-old boy during health surveillance visits. Which physical characteristics would be seen in both teenagers?

Increased shoulder, chest, and hip widths

The nursing instructor is presenting the basic physiologic changes in the woman which can occur during a pregnancy. The instructor determines the session is successful when the students correctly choose which change in the respiratory function during pregnancy as normal?

Increased tidal volume

A nurse is preparing a presentation for a group of new nurses about the potential for misuse of genetic discoveries and advances. Which aspect would the nurse most likely address?

Individual risk profiling and confidentiality

A father brings his 6-year-old son in to the doctor's office. He says his son just began first grade a week ago and, after attending the first day of classes, has refused to go to school each day since, complaining of stomach cramps and nausea. He says he has spoken to the boy's teacher, who says she did not notice anything unusual with her interaction with the boy, other than perhaps a bit of homesickness. After the nurse examines the boy and finds nothing wrong, the father asks how he should handle the problem. What action should the nurse recommend?

Insist that the child return to school.

The nurse is preparing to assess a client who is noted to have a history of dysmenorrhea. The nurse predicts the client will report which potential symptom during the examination?

Intermittent, sharp suprapubic pain

The nurse has determined that an 8-year-old girl is at risk for being overweight. Which intervention would be a priority prior to developing the care plan?

Interviewing the parents about their eating habits

Which element is a characteristic of therapeutic communication?

Is constructive

Which statement best explains the action of the lactational amenorrhea method (LAM) of fertility control?

LAM causes suppression of the ovulation process.

The nurse is performing an assessment of the reproductive system of a 17-year-old girl. What would alert the nurse to a developmental delay in this girl?

Lack of occurrence of first menstrual period

Inability to conceive can be very stressful on a couple. What is one psychological aspect of infertility?

Loss of intimacy

The nurse is explaining the latest laboratory results to a pregnant client who is in her third trimester. After letting the client know she is anemic, which heme iron-rich foods should the nurse encourage her to add to her diet?

Meats

The nurse informs the client that a diaphragm is an example of which type of contraception?

Mechanical barrier

What can the nurse educate a subfertile couple about to avoid the problem with lower sperm counts in the male partner?

Minimize actions that increase scrotal heat.

A woman is confused after finding out the ultrasound results predict a different due date for the birth of her baby. Which factor should the nurse point out is most likely the reason for the miscalculation of the fetal age?

Mistaking implantation bleeding for LMP

A male client visits a fertility clinic after one year of attempting unsuccessfully to impregnate his wife. What is a risk factor associated with male infertility?

No or few sperm cells produced

A nurse is caring for a hospitalized 10-year-old. What would be an appropriate activity for this child to meet the developmental tasks of this age group?

Participating in a craft project

A nurse is teaching a class about natural methods of birth control. Which group would the nurse identify as a good candidate for these methods of family planning? Select all that apply.)

Perimenopausal women Postpartal women

The nurse is preparing to teach a community class to a group of first-time parents. Which information should the nurse include concerning what the pregnant woman's partner may experience as a normal response?

Physical symptoms similar to the mother

A client is at 20 weeks' gestation and is scheduled for a fetal survey with ultrasound. The nurse teaches the client about the test The nurse determines that the teaching was successful when the client identifies that the ultrasound will reveal which information about the placenta? Select all that apply.

Placement of the placenta

What is the organ with a rich blood supply that supplies the developing organism with food and oxygen and carries away waste for excretion by the mother?

Placenta

Assessment of a pregnant woman reveals oligohydramnios. The nurse would be alert for the development of which condition?

Placental insufficiency

A 9-year-old with rheumatoid arthritis has difficulty moving her painful hands as well as her other joints. She refuses to participate in ordered physical therapy. What would be the best way for the nurse to make sure she continues to exercise her joints?

Play a game like "Simon Says" to introduce exercises.

A female client complains of irregular menses. On further assessment, the nurse observes that the client is obese and is developing hirsutism. Her diagnosis indicates an ovarian dysfunction with increased levels of testosterone. The nurse interprets these findings to suggest:

Polycystic ovarian syndrome.

A nurse is speaking with a 7-year-old boy in the hospital. The boy asks the nurse, "Where is the westroom? I need to use the westroom." The nurse recognizes that this developmental problem is which of the following?

Poor articulation

A 6-year-old child is learning how to draw insulin into a syringe and becomes frustrated, throwing himself on the floor. What is the best action by the nurse?

Provide positive reinforcement for the steps achieved, ignoring the temper tantrum

A mother tells the nurse she is having difficulty getting her 6-year-old to do chores. Based on the child's developmental level, what activity would be best for the nurse to recommend to the mother?

Putting books on shelf

The nurse teaches parents of adolescents that teenagers need the support of parents and nurses to facilitate healthy lifestyles. What should be a priority focus of this guidance?

Reducing risk-taking behavior

A woman in the third trimester of her first pregnancy expresses fear about the birth canal being wide enough for her to push the baby through it during labor. She is a petite person, and the baby seems so large. She asks the nurse how this will be possible. To help alleviate the client's fears, the nurse should mention the role of the hormone that softens the cervix and collagen in the joints, which allows dilation and enlargement of the birth canal. What is this hormone?

Relaxin

A nurse is caring for a client who is 8 months pregnant. Which instruction is the nurse most likely to give her?

Rest on the left side for at least 1 hour in the morning and afternoon.

A young newly married woman comes to the clinic and asks about ways to prevent pregnancy. When the nurse begins to talk about oral contraceptives, the client says that her religion does not allow oral contraceptives. What can the nurse recommend for this client?

Rhythm method

A nurse is counseling a pregnant client about Maternal Serum Marker Screening. She wants to go ahead with the test to rule out anencephaly. The client understands the risks associated with the test and is worried about what to do if the test shows a positive result even if the child is completely healthy. Which of the following nursing diagnoses would be most appropriate?

Risk for Anxiety related to false-positive test results

The nurse was teaching a college student how to properly take the prescribed combination oral contraceptive (COC). The nurse determines the session is successful when the client correctly chooses which instruction to follow when taking the COC?

Same time of day, each day

An 11-year-old female child is at the pediatrician's office for a well-child check-up. Which health screening would the nurse anticipate that the child would undergo today?

Scoliosis screening

A female patient who has been trying to conceive for 1 year comes to the fertility clinic and will be undergoing a subfertility evaluation. When discussing the components involved, which information would the nurse most likely include? Select all that apply.

Semen analysis of the partner Ovulation monitoring Tubal patency assessment

The child states, "I never want to be a nurse or a doctor." The nurse recognizes this statement as reflecting what level of communication?

Shared personal ideas and judgments

A young mother in a prenatal class asks the nurse why there is amniotic fluid in the uterus with her baby. Which functions of the amniotic fluid should the nurse point out to the client? Select all that apply.

Shields the fetus against pressure or a blow to the mother's abdomen Protects the fetus from changes in temperature

The community nurse is preparing a presentation for a health fair illustrating successful pregnancies. Which compotent should the nurse prioritize as the most critical to ensure a postive psychological experience with the pregnancy by the mother?

Social support

A female patient had a diagnostic laparoscopy and is now reporting a sharp pain in her shoulder. What response is a priority by the nurse?

Some of the carbon dioxide that was used to insufflate the abdomen has likely escaped under the diaphragm and this will resolve on its own.

A client who suspects she is pregnant asks the nurse about the accuracy of home pregnancy tests. The nurse would tell the client that:

Some of the home pregnancy tests can detect the presence of hCG within one day of the woman's missed period.

The nurse is teaching a nutritional class to a group of Asian adolescents who are lactose-intolerant. Which food should the nurse point out will help these adolescents consume the calcium they need for proper nutrition?

Soybean curd

A 15-year-old client with type 1 diabetes has been noncompliant with his dietary regimen. When educating the teen, what is the most important thing the nurse can do to allow the teen to be in control and involved in the decision-making process?

Speak directly to the teen and consider his input in the decisions about care and education.

A young couple is having difficulty getting pregnant. The nurse is preparing the couple for the initial tests to determiine their fertility. When asked by the couple why they need to start with a sperm analysis, what will be the nurse's best response?

Sperm analysis is one of the easiest tests to complete.

A young couple is having difficulty getting pregnant. The nurse is preparing the couple for the initial tests to determine their fertility. When asked by the couple why they need to start with a sperm analysis, what will be the nurse's best response?

Sperm analysis is one of the easiest tests to complete.

True adolescence is said to begin when what occurs?

Sperm is produced in the male.

The nurse is teaching a 6-year-old girl and her mother about home care for an eye infection. Which of the communication techniques would be least effective with this child?

Standing beside the child when doing the teaching

A 6-year-old male has come to the clinic for a routine well-child visit. The nurse, after noting in the medical record that this child has followed basic growth and development standards, anticipates documenting which common assessment finding?

Swayed back

An 18-year-old reveals that she has a nipple ring and is looking to get a tattoo in the next few months. What is the most important thing that the nurse can teach her at this time?

Tattooing carries risks such as infection, disease, and nerve damage.

The nurse is helping the parents and their underweight adolescent collaborate on planning a healthy menu. The nurse should be aware of which nutritional requirement of adolescents?

Teenagers have a need for increased calories, zinc, calcium, and iron for growth.

The nurse is preparing to participate in a community discussion on the needs of the adolescents in the local school. The nurse should point out which goal is the primary concern for these young individuals as the committee makes plans?

Teens are busy developing their own personal identity.

When describing the characteristics of the amniotic fluid to a pregnant woman, which would the nurse include?

The amount gradually fluctuates during pregnancy.

The nursing instructor is leading a class discussion on the various aspects of adolescents. The instructors determines the class is successful after the students correctly choose which milestone as the beginning of adolescence?

The beginning of puberty

The nurse midwife is performing a pelvic examination on a client who came to her following a positive home pregnancy test. The nurse checks the woman's cervix for the probable sign of pregnancy known as Goodell's sign. Which description illustrates this alteration?

The cervix softens.

The nurse is assessing developmental milestones for a 5-year-old girl. Which of the following activities is normal?

The child can button a shirt

The nurse is performing a physical examination of an 11-year-old girl. What observations would be expected?

The child has grown 2.5 inches since last year.

Which behavior best demonstrates an example of Erikson's developmental task for the school age child?

The child spends a rainy day putting together a puzzle of a large jungle animal.

A pregnant client in her first trimester states, "I think I must be having a miscarriage. I have sharp pains in my lower abdomen sometimes!" What does the nurse understand is happening to this client?

The client is experiencing stretching of the round and broad ligaments.

A Maternal serum alpha-fetoprotein (MSAFP) test reveals an hCG level of 2.5 MoM. The nurse interprets these results as indicating which of the following?

The fetus is at greater risk for Down's syndrome

A pregnant client asks the nurse how the fetus breathes if it is floating in the amniotic fluid. The best explanation by the nurse is:

The fetus receives oxygen through the umbilical vein.

Which vaccine is safe to give in pregnancy?

The flu vaccine after the first trimester

The nurse is assessing a pregnant client in her third trimester who is reporting a first-time occurrence of constipation. When asked why this is happening, what is the best response from the nurse?

The intestines are displaced by the growing fetus.

While conducting a class for a group of pregnant women in their first trimester, one of the woman asks, "How is my baby's sex determined?" Which statement would the nures include in the response? Select all that apply.

The male gamete is responsible for sex determination. The Y chromosome is smaller and contains genes for maleness.

The nurse is assessing a 27-year-old woman pregnant for the first time during her initial visit to the obstetric clinic. The patient is currently at 8 weeks' gestation. During the assessment, the patient reports that she is experiencing sharp stabbing pains in her lower abdomen, especially when she changes position. How would the nurse document this finding?

The patient is having growing pains

A male client asks the nurse to explain which structure is cut during a vasectomy. What response should the nurse give the client?

The vas deferens

A woman trying to conceive has been prescribed metronidazole (Flagyl) for bacterial vaginosis. What should the nurse caution this patient while taking this medication?

This drug can be teratogenic early in pregnancy and shouldn't be continued if a pregnancy is suspected.

The nurse is preparing a teaching session for a client considering tubal ligation. Which factor should the nurse prioritize in this session?

This is a permanent and irreversible procedure for birth control.

A young woman says she needs a temporary contraceptive but has a latex allergy. She mentions that she has a papillomavirus infection. Also, she says she is terrible about remembering to take pills. Which method should the nurse recommend?

Transdermal contraception

While talking with a pregnant woman who has undergone genetic testing, the woman informs the nurse that her baby will be born with Down syndrome. The nurse understands that Down syndrome is an example of a:

Trisomy numeric abnormality.

Couples who engage in coitus daily, hoping to cause early impregnation, may actually have more difficulty conceiving than those who space coitus to every other day.

True

Morning sickness is associated with rising levels of human chorionic gonadotropin (hCG) and progesterone.

True

Pregnant women seem to be more susceptible to carpal tunnel syndrome than others.

True

Smoking may be a cause of ectopic (tubal) pregnancy.

True

Suicide is so common in adolescents it ranks as the third leading cause of death in the 10- to 24-year-old age group.

True

During an examination, a client who is 32 weeks pregnant becomes dizzy, lightheaded, and pale while supine. What should the nurse do first?

Turn the client on her left side.

A 33-year-old pregnant client asks the nurse about testing for birth defects that are safe for both her and her fetus. Which test would the nurse state as being safe and noninvasive?

Ultrasound

A pregnant woman is scheduled to undergo an amniocentesis. When explaining this test to the client, the nurse would also include information about which test being done at the same time?

Ultrasound

The nurse is assessing a primigravida woman at a routine prenatal visit. Which assessment finding is reinforcing to the client that she is definitely pregnant?

Ultrasound picture of her fetus

The nurse teaches a primigravida client that lightening occurs about 2 weeks before the onset of labor. What will the mother likely experience at that time?

Urinary frequency

A nurse is preparing to start an intravenous (IV) line in a child with severe pneumonia. The nervous child asks the nurse to wait until later to do the procedure. What is the best option for the nurse?

Use a firm, positive, confident approach when starting the IV.

The nurse is preparing a 38-year-old pregnant woman for an amniocentesis to determine if there are any congenital disorders or fetal abnormalities present. Which of the following is a nursing consideration before and during the testing?

Use an external fetal heart monitor to ensure fetus is not in distress.

A 22-year-old female client asks about forms of chemical barrier contraception. The nurse tells the client that which is a form of a chemical barrier contraceptive?

Vaginal foam

The mother of an adolescent has called to talk to the pediatric nurse over concern that her daughter is not getting proper nutrition now that she has started following a vegetarian diet. Which response should the nurse prioritize for this mother?

Vegetarian diets can be healthy; ensure she includes whole-grain products, legumes, nuts, seeds, and soy dairy substitutes.

A client who has just given a blood sample for pregnancy testing in the health care provider's office asks the nurse what method of confirming pregnancy is the most accurate. The nurse explains the difference between presumptive symptoms, probable signs, and positive signs. What should the nurse mention as an example of a positive sign, which may be used to diagnose pregnancy?

Visualization of the fetus by ultrasound

A pregnant client in her second trimester informs the nurse that she needs to travel by air the following week. Which precaution should the nurse instruct the client to take during the flight?

Wear support hose.

A 7-year-old seen in the clinic for a routine well-child visit is noted on assessment to have gained 5 lb (2.26 kg) and grew 5 in (12.70 cm) over the past year. The nurse determines this child is within which parameters?

Weight is within expected range and height is above expected range.

A client in her third trimester reports sleeping poorly: sleeping on her back results in lightheadedness and dizziness and lying on her side results in no sleep. Which suggestion for sleeping should the nurse prioritize for this client?

With a pillow under her right hip

The nurse is discussing the insulin needs of a primaparous client with diabetes who has been using insulin for the past few years. The nurse informs the client that her insulin needs will increase during pregnancy based on the nurse's understanding that the placenta produces:

hPL, which deceases the effectiveness of insulin.

Semen analysis has been ordered for the partner of a client who has been unable to become pregnant. What instructions should the nurse provide to the partner?

"Bring the sample to the lab immediately after you collect it."

The nurse is assessing a male client who is concerned about his ability to produce enough sperm to have a child. He tells the nurse, "I have had some issues in my younger years." What questions would be important for the nurse to ask this client? Select all that apply.

"Do you use drugs or use alcohol excessively? "Are you exposed to X-rays or other radioactive substances?" "Have you ever had any type of trauma or surgery on or near your testicles?"

A pregnant client in the first trimester asks the nurse about taking medications while she is pregnant. She tells the nurse that she heard that it can be harmful to the fetus if medications are taken at certain times during pregnancy. What is the best response by the nurse?

"Exposure to certain substances during the embryonic phase may be harmful to the developing fetus."

A male nurse is meeting with a group of 12-year-old boys to discuss expected bodily changes. After one of the boy's says, "My older brother told me my bed might be wet and that means I had a wet dream. Is that true?" What is the best response from the nurse?

"Having wet dreams indicates that your body is going through a process of maturing."

The nurse is caring for a 13-year-old girl. The child has been identified as overweight with no underlying psychological or secondary causes. The nurse is reviewing the child's weight-loss progress and nutrition at a follow-up visit. What finding indicates a need for further discussion and teaching?

"Her goal is to be a size smaller by our vacation in two weeks."

A male nurse is meeting with a group of high school boys to discuss various health topics. After the session on testicular self-exam, the nurse determines the session is successful when one of the students responds with which comment?

"I am almost 15 now, so that means I could possibly get this disease."

The school nurse is teaching a health class on nutrition with some adolescents. Which comment by a student should the nurse prioritize and provide more teaching?

"I am feeling so fat. I think I need to exercise a few extra hours today."

Which statement made by a client with a chlamydial infection indicates understanding of the potential complications?

"I need to treat this infection so it doesn't spread into my pelvis because I want to have children some day."

The nurse is caring for a couple who are trying to get pregnant and have not been able to for over a year. The nurse explains to the couple that diagnostic testing usually begins on the male partner, as these tests are easier. The couple asks what kind of problems a man can have that can cause infertility. What should be the nurse's response?

"Men can have problems that increase the temperature around their testicles, which decreases the quality of their semen."

The nurse is teaching a group of caregivers of school-age children about the importance of setting a consistent bedtime for the school-age child. Which statement made by a caregiver indicates an understanding of the sleep patterns and needs of the school-age child?

"My child sleeps between 11 and 12 hours a night."

A client is scheduled to have in vitro fertilization (IVF) in 1 week. Which statement made by the client indicates that she needs further teaching?

"The primary care provider will transfer the egg and sperm into the fallopian tube where the egg will become fertilized."

A client reports occasional headaches. She wants to know what she can take to alleviate the discomfort. What would be the best response by the nurse?

"The safest medication to take for your headaches during your pregnancy would be acetaminophen."

The nurse is teaching the parents of a 12-year-old boy about common approaches when raising an adolescent. Which instruction is most important?

"Try to be open to his views."

The parents of an 11-year-old child ask the nurse for suggestions to promote good nutrition for their child. Which response by the nurse would be most appropriate?

"Use whole-grain or enriched breads and cereals."

A 25 year old having a yearly check-up informs the nurse that she has stopped taking birth control pills because she and her husband want to start a family. She states, "I know I will get pregnant right away, because my mother was very fertile." What is the nurse's best response?

"Women who used oral birth control pills may have difficulty becoming pregnant for several months after discontinuing them."

A 6 year old says to the nurse, "I am scared that my mom and dad won't be back before I have to have a shot." What statement by the nurse gains more information by use of reflecting?

"You are scared?"

A parent brings a 6-year-old to the clinic and informs the nurse that the child is tired all the time even though the child sleeps 7 to 8 hours each night. What is the best response by the nurse?

"Your child should be getting 11 to 12 hours of sleep per night with some quiet time after school."

Which situation may be enhanced by the use of drawings to elicit feelings from a child?

A 4-year-old scheduled for surgery

Which of the following changes, with highest priority, should the nurse teach a pregnant patient to report to the health care provider as soon as possible?

Abdominal pain coming and going during the third trimester

When caring for hospitalized teens, nurses should choose their words and actions carefully since adolescents typically are concerned about:

Appearing out of control of the situation and/or themselves

While treating a minor playground injury for an 8-year-old girl, the school nurse discovers that the injury was the result of bullying. What should be the nurse's first action?

Assess the situation with the help of the school staff and parents.

A patient in her third trimester comes in for a routine prenatal visit. The nurse places her in a comfortable position and attaches the tocodynamometer and ultrasound monitor to the patient's abdomen. What is the purpose of this test?

Assesses fetal well-being

What should be the first step in developing a teaching plan for a 9-year-old who needs education about a gluten-free diet for the treatment of celiac disease?

Assessing the child's current level of understanding

During an exam, the nurse notes that the blood pressure of a client at 22 weeks' gestation is lower, and her heart rate is 12 beats per minute higher than at her last visit. How should the nurse interpret these findings?

Both findings are normal at this point of the pregnancy.

Mercury occurs naturally in the environment, including waterways. Bacteria in the water convert mercury to methylmercury, which is absorbed by fish low on the food chain and becomes concentrated in larger, longer-living predatory fish at the top of the food chain. What can mercury exposure during pregnancy cause?

Brain and neurologic abnormalities in the fetus

A woman comes to the clinic complaining of irregular contractions lasting less than 30 seconds and occurring no more frequently than 5 times in 1 hour. She is afraid of losing the pregnancy. She is at 26 weeks of gestation with her first child. What is most likely happening to this woman?

Braxton Hicks contractions

The nurse is examining a woman who came to the clinic because she thinks she is pregnant. Which data collected by the nurse are presumptive signs of her pregnancy? Select all that apply.

Breast changes Amenorrhea Morning sickness

A young patient comes to the clinic and requests birth control pills to prevent a pregnancy. She tells the nurse that she is worried that she will get mixed up on what days to take the pills. Which would be the best type of pills to prescribe this patient?

COCs packaged with 28 pills

A 6-year-old reports pain in the stomach upon eating. The nurse replies, "Let me see if I have this right. Every time you eat anything, you get a pain in your tummy?" The nurse is using which technique of therapeutic communication?

Clarifying

A nurse is talking with a 10 year old who is saying that his "stomach has been hurting for several days and is worse when he drinks milk." The nurse asks the child, "Let me be sure I understand. The pain gets worse when you drink milk?" What type of therapeutic communication technique is the nurse using?

Clarifying

A nurse is assigned to care for a 7-year-old with cystic fibrosis. The child wants to show the nurse his collection of baseball cards. The nurse understands that the collection of objects is common in this age group and is known as what type of thinking?

Classification

While assessing a client's breast during the third trimester, which finding would the nurse expect?

Colostrum from the nipples

Which effect would the nurse identify as a normal physiologic change in the renal system due to pregnancy?

Dilation of the renal pelvis

A pregnant woman undergoes a triple/quadruple screen at 16 to 18 weeks' gestation. What would the nurse suspect if the woman's level is decreased?

Down syndrome

The nurse is assessing a young female who just found out she is pregnant. She is is now reporting vague abdominal discomfort. After noting the client has a history of PID, the nurse predicts the health care provider will give priority to ruling out which situation?

Ectopic pregnancy

A nurse is taking care of a teenager who reports involuntary discharge of semen while sleeping. The nurse observes that the client is confused and does not know about the pubertal changes that are taking place in his body. Based on this information, what is the highest priority area that should be reviewed with the client?

Educate the client about the changes that occur during adolescence.

Prenatal testing is used to assess for genetic risks and to identify genetic disorders. In explaining to a couple about an elevated alpha-fetoprotein screening test result, the nurse would discuss the need for:

Further, more definitive evaluations to conclude anything.

What teaching points would a nurse provide for families of school-aged children to help prevent substance abuse? Select all that apply.

Give the child "what if" examples to situations they may face. Set firm rules regarding alcohol and other drug usage and discuss consequences associated with breaking the rules. Encourage decision-making and discuss family values.

The nurse is performing an assessment of a woman who has come to a health care facility for a diagnosis of pregnancy. The women is positive for breast changes, nausea, and amenorrhea. On physical exam, it is noted that the client has softening of the cervix. How should the nurse document this in her notes?

Goodell sign

In the first trimester, sharp pains in the lower abdomen are common. They are not repetitive but often are associated with position changes or fetal movements. What are these indicative of?

Growing pains

The parents of a 16-year-old male are worried about recent changes in his behavior, ignoring his schoolwork and sports, and spending almost all of his free time interacting with his girlfriend. Which suggestion should the nurse point out would best address this situatio

He has developed his own identity by now; being able to establish close relationships with girls is important preparation for all of his adult relationships. They should honor his need to be with, or talk to, his girlfriend as long as he has completed his schoolwork for the day.

What is the most positive benefit that effective therapeutic communication has in the nurse-client relationship?

Helps develop trust between nurse and the child.

A nurse assessing the laboratory results of a pregnant client in her second trimester notes that she has a hemoglobin level of 11 gm/dL. What will the nurse interpret this finding to most likely indicate?

Hemodilution of pregnancy

During a well-child visit, the mother of a 10-year-old mentions to the nurse that they are thinking about getting a trampoline for their daughter. When responding to the mother, the nurse would need to keep in mind which of the following?

Home trampoline use should be discouraged

A patient presents at the emergency department. During the assessment, the nurse notes the following: Patient is a 22-week primipara, age 25, pulse 82, BP 110/76, temp 38.3°C. The patient is diagnosed with pyelonephritis. What would be the treatment of choice?

Hospitalization and intravenous antibiotics

When describing genetic disorders to a group of couples planning to have children, the nurse would identify which as an example of an autosomal dominant inheritance disorder?

Huntington disease

Nursing care is evident in the community when a patient needs to receive home care. One of the disadvantages of remaining in a hospital rather than receiving care at home is

Increased chance for infection

A student nurse is preparing for a presentation which will illustrate the various physiologic changes in the woman's body during pregnancy. Which cardiovascular changes up through the 26th week should the student point out?

Increased pulse rate and decreased blood pressure

A nurse is teaching a group of patients experiencing secondary subfertility. Which of the following would the nurse include as things that can negatively affect spermatogenesis? (Select all that apply.)

Increased scrotal heat Trauma to the testes Drug use Excessive alcohol use X-ray exposure

A 28-year-old primigravida client with diabetes mellitus, in her first trimester, comes to the health care clinic for a routine visit. The client reports frequent episodes of sweating, giddiness, and confusion. What should the nurse tell the client about these experiences?

Increased secretion of insulin occurs in the first trimester

A nurse is preparing a presentation for a group of nurses about genetic discoveries and advances. When discussing this topic, the nurse would most likely include which topic area related to the potential for misuse?

Individual risk profiling and confidentiality

A young woman comes into the clinic stating that she is pregnant and does not want to have a baby. After counseling, the woman decides to terminate the pregnancy by having an abortion. The nurse explains that the procedure will be what type of abortion?

Induced abortion

During a routine antepartal visit, a pregnant woman says, "I've noticed my gums bleeding a bit since I've become pregnant. Is this normal?" The nurse bases the response on the understanding of which effect of pregnancy?

Influence of estrogen and blood vessel proliferation

A woman is using depot medroxyprogesterone acetate (Depo-Provera) as a method of birth control. Which side effect would the nurse most likely include as common?

Irregular menstruation

The nurse is talking with parents of a depressed 16-year-old boy. Which question is of the most importance?

Is there a gun in your home?

A woman is undergoing testing which provides a picture-like analysis of the number, form, and size of the woman's chromosomes. The nurse identifies this as:

Karyotyping.

A client calls to cancel an appointment for the first prenatal visit after reporting a home pregnancy test is negative. Which instruction should the nurse prioritize?

Keep the appointment.

The nurse is meeting with a group of caregivers of adolescents. Which example should the nurse point out is most effective for the caregiver to support the adolescent?

Let them choose their hairstyle, even though it may not look the best for them.

The school nurse is planning to teach a segment on smoking during the freshman health classes. The nurse is aware that this needs to be a forum rather than a lecture. Which of the following techniques will also help deliver a "don't smoke" message?

Listening to all comments nonjudgmentally

A client in her second trimester of pregnancy visits a health care facility. The client frequently engages in aerobic exercise and asks the nurse about doing so during her pregnancy. Which precaution should the nurse instruct the pregnant client to take when practicing aerobic exercises?

Maintain tolerable intensity of exercise.

A woman is taking vaginal progesterone suppositories during her first trimester because her body does not produce enough of it naturally. She asks the nurse what function this hormone has in her pregnancy. What should the nurse explain is the primary function of progesterone?

Maintains the endometrial lining of the uterus during pregnancy

A client reports that her last menstrual period was on August 1. Using Naegele's rule, what would the nurse anticipate as the client's due date?

May 8

The nurse is assessing a pregnant woman. What assessment would suggest that the client's heart has been pushed upward and to the left by pregnancy?

Murmurs

A woman who has been trying to conceive has been diagnosed with multiple myomas of the uterus. What option is available for the woman to be able to conceive?

Myomectomy

The nurse has given an adolescent a pamphlet about sexually transmitted infections (STIs). Which documentation by the nurse shows that the teaching has been effective?

Named two methods used to reduce the risk of STIs

A client is beginning to take a combined oral contraceptive. Which of the following side effects will the nurse caution the client might be expected? Select all that apply.

Nausea Headache Breast tenderness

The nurse is promoting learning and school attendance to a 13-year-old girl. Which factor will affect the child's attitude most?

Peer group behaviors and attitudes

When providing anticipatory guidance to a group of parents with school-aged children, what would the nurse describe as the most important aspect of social interaction?

Peer relationships

After teaching a class on the stages of fetal development, the nurse determines that the teaching was successful when the group identifies which stages? Select all that apply.

Pre-embryonic Embryonic Fetal

A client in her third trimester of pregnancy visits the health care center and asks why she is constipated. The nurse would include which most likely cause when responding to the client?

Pressure on intestine by the growing fetus

Amanda's menstrual period is two weeks late. She has been feeling tired and has had bouts of nausea in the morning. What classification of pregnancy symptoms is Amanda experiencing?

Presumptive

Assessment for surfactant level via LS ratio in the amniotic fluid is a primary estimation of fetal maturity. The purpose of surfactant is to:

Prevent alveoli from collapsing on expiration.

A nurse is teaching children about proper nutrition and shows them the USDA "My Plate." The nurse discusses the importance of diet and daily exercise. This is an example of which type of health care?

Preventive

A nurse is driving to visit a home care patient when she notices a car following and several youths lingering on the corner. The nurse feels uneasy. Which is the best option for the nurse to take in this situation?

Proceed to the nearest police station for an escort.

A nursing instructor informs a student to record her interaction with a child who has osteogenesis imperfecta to determine if the conversation was therapeutic. What form of evaluation is this?

Process recording

A 3 month old has been scheduled for surgery at 10 am the next day. During preoperative teaching, the baby's mother interrupts the nurse and says that she does not know how she will make it if she has to keep her baby NPO after midnight. The nurse informs the mother that the infant will have to be NPO for as little as 4 hours. This is due to which of the following reasons?

Risk for dehydration

Which of the following is NOT a common discomfort of pregnancy?

Scotomas

A nurse is introduced to her 8-year-old patient with sickle-cell crisis. The child states, "I'm Johnny and I am 8." What level of communication does this patient display?

Second Level: Fact Reporting

The nurse is preparing a presentation for a local parent-teacher organization about the growth and development of school-age children. Which of the following would the nurse include?

Secondary sex characteristics are often embarrassing for both sexes.

A couple comes to the clinic stating that they have one child who is 4 years old, but they have been trying to have another child for 1 year without success. What type of subfertility would this be considered?

Secondary subfertility

A couple informs the nurse that they have not been able to conceive for the last year. During assessment, the nurse learns that the couple has a 3-year-old daughter and 7-year-old son. They report having had no problems getting pregnant the other times. Which type of fertility is this couple exhibiting?

Secondary subfertility

Spermatozoa begin their cellular division in what part of the male reproductive system?

Seminiferous tubules

A couple comes in and tells the nurse that they have been trying diligently to conceive for the last 3 months and are subfertile. What should the nurse explain to the couple about the definition of subfertility?

Subfertility is said to exist when a pregnancy has not occurred after at least 1 year of engaging in unprotected coitus.

A 16-year-old client has been hospitalized 100 miles from home for 1 week to repair a fractured patella suffered in a skateboarding accident. She was cheerful and chatty when she first arrived, but the nurse notes in recent days she has become increasingly quiet and seems lonely. Which nursing intervention should the nurse prioritize for this client?

Take her to the teen lounge so she can meet other teens, use a phone, and check her e-mail.

An 8-year-old is scheduled to have a tonsillectomy and adenoidectomy in 2 weeks. What intervention can the nurse provide to help the child and family adjust to the hospitalization?

Take the child on a tour of the facility and surgical suite and explain what to expect preoperatively and postoperatively.

The nurse is providing education to an adolescent prescribed oral tetracycline. What statement should the nurse include in this teaching?

Take the medication on an empty stomach.

A pregnant client is visiting the clinic and complains about the tiny, blanched, slightly raised end arterioles on her face, neck, arms, and chest. The nurse should explain that these are normal during pregnancy and referred to as which of the following?

Telangiectasias

A couple has just undergone a semen analysis. The semen sample was visualized under the microscope, and the nurse tells the couple that they must return to do another semen analysis. The man states that he can return later in the week. What is the best answer the nurse can provide?

The analysis will need to be repeated after 2 or 3 months because spermatogenesis is an ongoing process, and 30 to 60 days are needed for sperm to reach maturity.

A 24-week pregnant client calls the clinic crying after a prenatal visit, where she had a pelvic exam. She states that she noticed blood on the tissue when she wiped after voiding. What initial statement by the nurse would explain this finding?

The cervix is very vascular during pregnancy, so spotting after a pelvic exam is not unusual.

Parents of an 11-year-old boy are bewildered that their son was caught stealing. The nurse best supports the parent by explaining which?

The child had a strong desire to own the item with little other way of obtaining it.

Which action by a 7-year-old child best demonstrates his or her development of decentration?

The child is able to recognize other people's point of view.

A nurse is talking with a 10-year-old and her parent about the current treatment plan for the child's asthma. The child stands behind the parent and does not ask questions or look at the nurse. What should the nurse consider the child's behavior could indicate?

The child may be shy and have some reluctance about communicating.

The nurse is conducting family teaching with the caregivers of school-age children on various topics. Which of the following statements is most accurate?

The child should have fluoride applied to their teeth at least twice a year.

The nurse knows that the school-age child is in Erikson's stage of industry versus inferiority. Which best examplifies a school-aged child working toward accomplishing this developmental task?

The child signs up for after-school activities.

A couple comes to the clinic and informs the nurse that they have been trying to conceive for 6 months with no success. The husband states that they have coitus at least 5 or 6 times per week to optimize their chance of success. What can the nurse educate the couple about frequency of coitus?

The couple should consider decreasing the frequency of coitus to 2 to 3 times per week to increase the male partner's sperm count.

The nurse is assessing a newborn infant for possible intrauterine growth restriction (IUGR). Which factor should the nurse analyze first to help determine if this diagnosis is accurate?

The ethnicity and stature of both the mother and father

An expectant mother in week 30 of her pregnancy reports to the nurse that she has been doing her kick counts several times a day and the fetus has been kicking at a rate of about 4 times per hour, on average. The nurse interprets this finding as suggesting which situation?

The fetus is not receiving enough nutrients

The nurse is teaching a pregnant teenager the importance of proper nutrition and adequate weight gain throughout the pregnancy. What is the best response when the client refuses to eat due to fear of possible weight gain?

The infant will be small and could have problems.

The nurse is providing prenatal care to a young couple who is pregnant with their first child. In what period of development would the nurse explain to the couple that most congenital defects would occur?

The period of the embryo

A group of nursing students are preparing a presentation depicting the fetal circulation. The instructor determines the presentation is successful when the students correctly illustrate which route for the ductus arteriosus?

The pulmonary artery to the aorta

A nurse is counseling a 17-year-old girl on the importance of contraception. The girl states that she is not really concerned about getting pregnant; she says that she will deal with it if and when it happens. Which point should the nurse emphasize with the client regarding the risks of unintended pregnancies? Select all that apply.

The woman is less likely to seek prenatal care. The woman is less likely to breastfeed. The woman is less likely to protect her fetus from harmful substances. The woman is less likely to complete high school or college. The woman is more likely to require public assistance.

The nurse enters a room and sees an 8-year-old who has just learned that he needs to have surgery staring into space with a sad expression. The nurse sits by the child and says, "You look so sad. Would you like to tell me about it?" The nurse is using which type of communication?

Therapeutic

Many couples attempt to influence the gender of the child they are trying to conceive by using special sexual positions, eating special food, or having intercourse at specific times. None of these are factors in the determination of the gender of the conceptus. What has research shown about gender determination?

There is a 50-50 chance of having a male or a female.

A client desires protection from unwanted pregnancies. However, the client does not enjoy sex when her partner wears a male condom. Also, the client experiences breast tenderness, headache, and nausea after taking oral contraceptives. Which method would be the most likely choice for the couple to help them enhance their sexual experience as well as prevent any side effects?

Transdermal contraceptive

Dilation and curettage is the method of abortion used when the gestational age of a pregnancy is still less than 13 weeks.

True

Green tea should be avoided during pregnancy

True

A pregnant woman states that she would like to take a tub bath but has heard from her aunt that this could be dangerous to the baby. Which instruction should the nurse give to the client?

Tub baths are fine unless you are unstable on your feet or are experiencing vaginal bleeding.

The nursing instructor is conducting a class discussion exploring the normal dentition progression of the school-aged child. The instructor determines the session is successful when the students correctly choose which factor as most likely occurring in 10-year-olds?

Two of the cuspid teeth have erupted.

A fertilized ovum is known as which structure?

Zygote

The nurse is assessing a pregnant client at her 20-week visit. Which breast assessment should the nurse anticipate documenting?

Darkened breast areolae

An anxious 12-year-old girl receives an injection from the nurse and sighs with relief when it is done. After a moment of reflection, the girl asks the nurse, "Is it hard to give someone an injection?" This girl's question is evidence that she has developed which cognitive skill?

Decentering

A 7-year-old is told that she will have to have her tonsils out during school break. Though disappointed, the child understands the problems with missing school. What type of behavior is the child displaying?

Decentration

A school-age child has demonstrated the ability to recognize several aspects of a problem at the same time and to understand cause and effect. The nurse should document the presence of which concept?

Decentration

A pregnant client comes to the prenatal clinic complaining of urinary frequency and lower back pain on the right, stating that this has never happened before. An exam validates the diagnosis of pyelonephritis. What factors would contribute to this condition?

Decreased peristalsis of urinary tract

The nurse should carefully screen a client who insists on using only combined oral contraceptives for which contraindication?

Deep vein thrombosis

A nurse is preparing to teach an 8-year-old recently diagnosed with diabetes how to give an insulin injection. Which is the best technique for the nurse to use?

Demonstration

A school health nurse is supporting a 15-year-old young woman with acne. What is a common myth related to acne in adolescent populations?

Diet plays a significant role in acne production.

A nurse is carrying on a conversation with a 7-year-old girl during an office visit. Which of the following is an example of the level of language development the nurse should expect in this child?

Difficulty understanding the concept of "half past" in reference to time

A woman in the 15th week of gestation is planning on terminating the pregnancy. The procedure done for second trimester terminations is which of the following?

Dilatation and extraction

A client in her second trimester of pregnancy has developed varicose veins and experiences leg cramps. Which suggestion would be most appropriate?

Elevate legs while sitting.

A 5-year-old girl tenses up when the nurse approaches to examine her. "Are you afraid?" the nurse asks her. The girl shakes her head in denial. As the nurse lifts the stethoscope to auscultate the girl's chest, however, the nurse notices that the girl tenses up again and grips the edge of the examination table tightly. "Oh—you are afraid of the stethoscope, aren't you?" the nurse replies. "It's okay—it doesn't hurt; see—reach out and touch it." Which communication technique is the nurse demonstrating here?

Empathy

A woman is 10 weeks pregnant and tells the nurse that this pregnancy was unplanned and she has no real family support. The nurse's most therapeutic response would be to:

Encourage her to identify someone that she can talk to and share the pregnancy experience.

A nurse is admitting a 16-year-old male to the floor for an appendectomy. How can the nurse prepare this client for hospitalization? Select all that apply.

Encourage him to keep his cell phone nearby to communicate with his friends. Interview the adolescent separately from the parent to allow expression of information that he may not be comfortable sharing in front of the parent. Provide privacy when client is changing into the hospital gown or going to the bathroom.

The nurse is preparing a presentation for a community health fair on the topic of helping school-aged children develop the self-confidence they will need to mature into responsible adults. Which suggestion should the nurse prioritize to caregivers to enable them to help their children in this area?

Caregivers should maintain consistent rules and expectations.

A woman is to receive methotrexate and misoprostol to terminate a first-trimester pregnancy. When preparing the teaching plan for this client, the nurse understands that misoprostol works by:

Causing uterine contractions to expel the uterine contents.

A 35-year-old female client who is unable to conceive is scheduled for a hysterosalpingogram to assess tubal patency. Which of the following would the nurse ensure before the client undergoes the hysterosalpingogram?

Client has just completed her menses

The school-aged child understands that a change in an object's shape does not necessitate a change in size. The nurse knows that this is an example of which type of cognitive development of the school-aged child?

Conservation

The nursing instructor is preparing a class presentation covering the various hormones and their functions during pregnancy. The instructor determines the class is successful when the class correctly matches which function with hCG?

Continues progesterone production by corpus luteum

A woman calls the prenatal clinic and says that she thinks she might be in labor. She shares her symptoms over the phone with the nurse and asks what to do. The nurse determines that she is likely in true labor and that she should head to the hospital. Which symptom is an indicator of true labor?

Contractions beginning in the back and sweeping forward across the abdomen

The school nurse is meeting with a 10-year-boy who is concerned about his weight. He reports he doesn't eat much candy but loves fruit, pasta, potatoes, and bread. Which suggestion should the nurse prioritize to help him maintain a healthy weight?

Encourage activities that will increase his physical activity.

Which example shows the nurse incorporating informal teaching while caring for a child?

Explaining the importance of adequate fluid intake while playing "tea party"

Which nurse best fosters a school-aged child's adaptation to hospitalization? The nurse who:

Explains procedures and shows the child equipment that will be used on them.

A nurse is caring for an 8 year old with a hearing impairment. What is the best way for the nurse to communicate with the child?

Face the child when speaking so he or she can follow the lip movement.

About 75% of pregnancies caused by in vitro fertilization end in spontaneous miscarriage.

False

The majority of pregnancy terminations are done in cases in which the pregnancy threatens the mother's life.

False

The nurse is providing contraception counseling to a perimenopausal woman who has had negative reactions to oral contraceptives in the past and would like a long-term, nonhormone-based method that has a high rate of success. Neither she nor her husband wants to undergo surgery, however. Which method should the nurse recommend?

Intrauterine device

When discussing contraceptive options, the nurse would recommend which option as being the most reliable?

Intrauterine system

A 15-year-old girl is in the hospital for surgery and is confined to bed. The nurse can tell that the client is nervous about being in the hospital. She tells the nurse that she feels "gross" and "on display" in her hospital gown. What should the nurse do to encourage a sense of autonomy and dignity related to the girl's body image?

Offer to assist the girl in washing her hair and let her pick the shampoo.

The nursing instructor is teaching a class on the various hormones necessary for a successful pregnancy and birthing process. The instructor determines the session is successful when the students correctly choose which hormone as being necessary after birth to ensure growth of the newborn?

Prolactin

A client tells the nurse that she has noticed an increase in her weight and fat deposits during the last year. The nurse reviews the client's chart and recognizes that the client is most likely going through puberty. Which nursing action is most appropriate at this time?

Provide reassurance that these are normal changes.

The nurse is explaining the ultrasound procedure to a pregnant client and mentions part of the purpose is to evaluate the amniotic fluid. When questioned by the client how the amniotic fluid helps the fetus, which functions should the nurse point out? Select all that apply.

Provides physical protection Helps regulates temperature Provides unrestricted movement Helps permit symmetrical growth

When interviewed by the school nurse, a 13-year-old adolescent female states she has a boyfriend and that her parents do not talk about sex with her. She says is confused about the facts and wants to know the truth. Which approach would best address this adolescent's concerns?

Sit down with her and openly discuss her concerns and questions in an honest, straightforward manner.

A pregnant woman asks the nurse about using herbal rememdies while she is pregnant. The nurse recommends that the woman talk with her health care provider about their use based on which understanding?

They are not rated with regard to safety during pregnancy.

The nurse is conducting a well-visit physical assessment on a 29-year-old female. The nurse determines the client is probably ovulating based on which condition of the cervical mucus?

Thin, slippery, and stretchy

The father of a 15-year-old daughter is concerned she is not getting adequate nutrition to play high school basketball. Her games are on Friday nights. Which suggestion should the nurse point out will best suit the needs of this adolescent?

Three daily meals that include choices from each of the food groups; Friday's lunch eaten around 2 p.m. with a small amount of fat and a somewhat larger than usual portion of complex carbohydrates.

The nurse is teaching a client about a vasectomy. The nurse determines the session is successful when the client correctly chooses which fact concerning the vasectomy?

Usually done as an office procedure

The nurse is assessing a young couple who desire to start a family and are questioning the nurse concerning various cautions to keep in mind. Which time period should the nurse point out that teratogenics pose the greatest risk and should be avoided?

Weeks 2 to 8

A nurse is instructing a client on birth control methods. The client asks about the cervical mucus method. When should the nurse tell the client she is fertile in relation to her mucus?

When it is thin, watery, and copious

A client is being prepared for artificial insemination. Which finding is the most suggestive to determine if the client is ovulating?

Change in the cervical mucus

An 8-year-old boy who says he wants to be a doctor when he grows up pleads with the nurse to let him put on his own band-aid after receiving an injection. The nurse agrees and watches as the boy very carefully lines the band-aid up with the mark left by the injection and applies it to his skin. Then he asks, "Did I do it right?" and waits eagerly for the nurse's feedback. The nurse recognizes in this situation the boy's attempt to master the primary developmental step of school age. What is that step?

Industry

A nurse overhears a 7-year-old calling her hospital roommate a vulgar name. What is the best action by the nurse?

Inform the child that talking to the roommate using that language is not acceptable behavior.

A nurse is talking to a mother concerned about her 5-year-old son. She informs the nurse that he eats only cereal and peanut butter every day and fears that he is not getting proper nutrition. The nurse reassures the mother that even though he is eating a limited variety of foods, he is likely getting enough nutrition. Which type of teaching is this nurse practicing?

Informal teaching

A nurse is providing a client with information on hormonal contraception. Which could the nurse use as an example of hormonal contraception?

Medroxyprogesterone

A client asks the nurse to describe the difference between menstruation and implantation bleeding. Which statements should the nurse include in the description? Select all that apply.

Menstruation occurs when the ovum is not fertilized. Implantation bleeding occurs when the blastocyst burrows into the endometrium. Menstruation occurs 14 days after ovulation. Implantation bleeding occurs 7 to 10 days after fertilization.

The school nurse is teaching parents risk factors for suicide in adolescents. What would the nurse discuss? Select all that apply.

Mental health changes History of previous suicide attempt Family disorganization Substance abuse

The nurse is assessing a pregnant client at her 12 weeks' gestation and the client reports some new bumps on the dark part of her nipples. What is the best response from the nurse when questioned by the client as to what they are?

Montgomery tubercles; secrete lubricant for the nipples

A father shares with a nurse that his 12-year-old son has begun ejaculating in his sleep and is deeply embarrassed about it. The father says he has tried to explain to his son that this is a normal occurrence in puberty, but the son is still humiliated by it. The nurse recognizes this occurrence as which of the following?

Nocturnal emissions

A student nurse is preparing an illustration for a health fair depicting the various types of twins. Which format will the student use to depict identical twins?

One egg and one sperm

A woman just gave birth to a healthy term newborn. Upon assessing the umbilical cord, the nurse would identify which structures as normal? Select all that apply.

One vein Two arteries

Which symptom may indicate a complication (not a common discomfort) in the second or third trimester of pregnancy?

Pain underneath the ribs on the right side

The client at 18 weeks' gestation states, "I feel a fluttering sensation, kind of like gas." The nurse understands that the client is describing what occurrence?

Quickening

Which teaching strategy would be most effective for a 7-year-old who is learning how to check blood sugars?

Re-demonstration

The nurse is developing a presentation for a group of young adult women about premenstrual syndrome. Which treatment options should the nurse point out as most appropriate? Select all that apply.

Reduction of caffeine intake Vitamin and mineral supplements NSAIDs

A couple comes to the clinic and states to the nurse, "I don't think we are ever going to be able to have children. We have been trying but have had no luck." What assessments does the nurse anticipate will be performed for this couple? Select all that apply.

Semen analysis Ovulation monitoring Tubal patency

The nursing instructor is teaching students about normal changes of pregnancy. The instructor talks about diastasis recti. What is the instructor presenting?

Separation of the muscles of the abdominal wall

The 6-year-old at a well-child office visit tells the nurse, "I can't play on teams because I am not as good at doing things as my big sister is." What suggestion should the nurse point out to the caregiver that will help increase the child's feelings of self-confidence?

Set up some play dates that include projects or activities in which the child can be successful.

A client makes an appointment with an obstetrician and assessment reveals positive Hegar and Chadwick signs. What should the nurse teach the client about these results?

She is probably pregnant, but this must be confirmed by other means

The nurse is meeting an 8-year-old girl with cancer and her family for the first time. Which will help to establish a therapeutic relationship with the child and family? Select all that apply.

Sitting at eye level with the child and parents Keeping both a relaxed posture and word flow Redirecting the conversation to maintain focus

A couple has chosen fertility awareness as their method of contraception. The nurse explains that the unsafe period for them during the menstrual cycle would be at which time?

Three days before and three days after ovulation

The nurse is assessing a pregnant woman on a routine prenatal visit. Which breast assessment finding will the nurse document as a normal and expected finding?

Tingling sensations and tenderness

Which change in the breasts should a nurse recognize as a normal change associated with pregnancy?

Tingling sensations and tenderness

A client in her third trimester is scheduled for a nonstress test. What is the purpose of the nonstress test for the client?

To determine the well-being of the fetus

The school nurse is teaching a class on sports injuries. What information is most important for the nurse to teach?

Vary the sports to prevent use injuries.

Positive signs of pregnancy are diagnostic, meaning nothing else can elicit that sign except pregnancy. What is the earliest positive sign of pregnancy?

Visualization of the gestational sac or fetus

A 12-year-old girl has recently begun menstruating and is well into puberty. She is visiting the doctor today for a routine physical examination. Which of the following findings should cause concern in the nurse?

Vulvar irritation

A mother calls the advance practice pediatric nurse practitioner about her 7-year-old daughter's dental hygiene. The daughter has had three cavities. She does not know what to do and asks the nurse for guidance. How should the nurse respond?

"Are you able to supervise her brushing?"

The nurse has just informed a client that her pregnancy test is positive and she will need further assessment to determine the complete status of the situation. Which initial emotional response does the nurse expect the client to exhibit?

Ambivalence

A nurse is teaching a young patient about the importance of good nutrition for wound healing. When the patient states, "I will eat more protein and take my vitamins so I will get better faster," this patient is demonstrating which type of learning?

Cognitive

A nurse is showing a 9 year old a video about why insulin is needed. The nurse then plans to discuss a booklet with the child on the same topic after the video. Of what type of learning is this an example?

Cognitive learning

A new mother asks the postpartum nurse if her baby is getting enough nourishment from breast-feeding within the first 24 hours following birth. The nurse would provide her what information?

Colostrum, which is the first milk produced, is rich in calories and protein that nourishes the infant well.

The nurse is meeting with a 36-year-old client who wishes to begin using contraceptives. The client reports being in a long-term, monogamous relationship, runs 2 miles per day, and smokes a pack of cigarettes each day. Which method will the nurse be least likely to suggest to the client?

Combination oral contraceptives

A pregnant client is concerned she may develop preeclampsia, so she has stopped adding any salt to her food and is now questioning the nurse about avoiding prepared foods. The nurse should point out some salt is very beneficial and can help prevent which negative outcome for her baby?

Congenital hypothyroidism

The nurse discusses various contraceptive methods with a client and her partner. Which method would the nurse explain as being available only with a prescription?

Diaphragm

The nurse is preparing a class for a group of adolescents about promoting safety. What would the nurse plan to include as the leading cause of adolescent injuries?

Motor vehicles

The nursing instructor is leading a discussion on school-aged children. The instructor determines the session is successful when the students correctly choose which factor as being a priority for the school-aged child?

Needs 10 to 12 hours of sleep per night

A nurse is preparing a patient for rhythm strip testing. She places the woman into a semi-Fowler's position. What is the appropriate rationale for this measure?

To prevent supine hypotension syndrome

A 16-year-old girl confides in the nurse that her parents are difficult to deal with and that it stresses her out. The nurse responds by saying, "You think that's stressful, you should see some of the patients I have to deal with in here!" Which barrier to communication is this nurse demonstrating?

Topping up

A client in her third trimester reports to the nurse shortness of breath when sleeping. The nurse informs the client that this is normal and occurs because the growing fetus puts pressure on the diaphragm. Which measure should the nurse suggest to help alleviate this problem?

Use extra pillows.

A nursing instructor is teaching about the different types of contraception and identifies a need for further instruction when one of the students states which of the following?

"Diaphragms help to prevent STDs."

A woman has recently found out that she is pregnant. She comes to the nurse with complaints about breast tenderness and fatigue. What is the best response from the nurse?

"That must be hard for you. Let's think of some ways to decrease this discomfort you are feeling."

A 38-year-old woman comes to the clinic to discuss discontinuing use of her birth control pills and attempting conception. She says to the nurse, "I know a lot of women who have waited later to have children and don't have any problem." What is the best response by the nurse?

"Women in their late 30s who use oral contraception may face a delay in conception for several months, because the body has to return to normal function before conception is possible."

A client asks the nurse about rate of fertility for women older than 35 years. Which of the following responses from the nurse is accurate?

A woman's fertility peaks in her 20s, declines gradually until age 35, and then rapidly declines until it ends.

The nurse is assessing a 52-year-old perimenopausal female who is concerned about the changes occurring in her body. When questioned about the most serious changes, which effect should the nurse point out?

Bone mineral density decreases

A client wants to know if she can engage in intercourse during pregnancy. Which information should the nurse confirm to ensure that sexual intercourse or orgasm is not contraindicated in the client? Select all that apply.

Client is not at risk for preterm labor. Client does not face a risk of threatened abortion.

The nurse in an emergency department is assessing a 17-year-old girl who is complaining of a gap in her memory of the previous night. She reports after drinking only one drink that she felt dizzy and disoriented. She woke up in a friend's bed but has no recollection of getting there. What action should the nurse prioritize for this client?

Complete a rape kit to rule out possible sexual abuse while she was blacked out.

A client in the clinic requests birth control pills to prevent pregnancy. Upon taking a sexual history, the nurse realizes that the client has multiple partners. Which type of contraception would be the best for the nurse to suggest?

Condoms

The nurse is conducting a routine well-visit on a 43-year-old client who is concernd about developing breast cancer, although no family members have experienced it. What is the best advice for this client at this time?

Conduct regular breast self-examination.

A nurse is counseling a woman who is in the office for her second prenatal visit. The client says that since her last visit, she has been able to cut her smoking from a pack a day to only two or three cigarettes a day. Which of the following would be the most appropriate response for the nurse to give her?

Congratulate her and encourage her to keep it up

The nursing student is preparing a presentation which will depict the pre-embryonic stage. The instructor determines the student has correctly illustrated the process by putting the stages in which order? Use all the options.

Formation of the zygote Cleavage Formation of the morula Formation of blastocyst Implantation

A client is having her vital signs and weight taken and recorded at a prenatal visit. She is in her second trimester at 23 weeks' gestation. Her weight gain in the first trimester was 2 pounds and she has currently gained 14 pounds overall. What is the nurse's interpretation of this data?

Her weight gain in the first trimester is less that expected but she has caught up and her weight gain is good.

A client comes to the clinic for a medical termination of pregnancy. Which potential complications should the nurse point out are possible with this type of procedure? Select all that apply.

Incomplete abortion Prolonged bleeding

A nurse is providing care to a pregnant woman in her first trimester who has come to the clinic for a follow up visit. During the visit, the nurse teaches the woman about some of the changes that she will be experiencing during her pregnancy. Which information would the nurse include when describing changes in the breast?

Montgomery's tubercles become more prominent.

A nurse is teaching a 7 year old what to expect during an upcoming tonsillectomy. In this situation related to teaching, which of the following is the encoder?

Nurse

A woman in her 15th week of pregnancy is about to undergo amniocentesis. Which nursing intervention should be made first?

Obtain a signed consent form.

A young woman is newly married and is seeking advice on contraception. She is in a monogamous relationship and would like a temporary contraceptive, as she plans to have children in the future. Her husband dislikes the feel of condoms. Also, she mentions that she typically experiences dysmenorrhea and has a history of recurrent urinary tract infections. Which method should the nurse recommend?

Oral contraceptive

What are methods for delivering hormonal contraception? Select all that apply.

Orally Transdermally Vaginally Implantation Injection

A woman has just been prescribed clomiphene citrate to stimulate ovulation. Which possible effect should the nurse warn the woman about?

Overstimulation of the ovary resulting in potential multiple births

The nursing instructor is preparing an illustration which will point out the various functions of the placenta during the pregnancy. Which hormones should the instructor point out are secreted by the placenta during the pregnancy? Select all that apply.

Progesterone Estrogen Human chorionic gonadotropin

What effect does progesterone have on normal gallbladder function?

Progesterone interferes with gallbladder contraction, leading to stasis of bile.

A pregnant client who is beginning her third trimester asks the nurse why she feels like she is sometimes having labor contractions. The nurse would explain that:

She is having "practice" contractions called Braxton Hicks contractions and they are normal.

A nurse is talking with a 9-year-old about a procedure that will be done in the morning. The child is expressing fear. What listening skills does the nurse exhibit that makes it clear the nurse is actively listening? Select all that apply.

Sitting at the level of the child Nodding in response to comments the child makes Maintaining eye contact while the child is talking

The nurse in a primary care clinic and a client have come to see the primary care provider because the client is getting married and wants to have a prenuptial gynecologic examination. After the exam, the nurse asks about the woman's plan to have a family. The woman responds, "Why do I have to plan for a family?" What would be the nurses's best response?

The couple can control the time between births when they plan their family.

A patient is to have an amniocentesis with ultrasound. What does the nurse explain to the patient that amniocentesis can determine? Select all that apply.

Whether the fetal lungs are mature enough to support respiration outside of the womb. For genetic testing.

What action by a parent would be least likely to foster development of self-confidence in a school-age child?

Comparing the child to an older sibling regarding academic achievements

A female client who comes to the clinic for a visit is diagnosed with Turner's syndrome. The nurse would expect the client to experience which of the following?

Compromised or absent fertility

The school nurse is presenting information to a group of high school students who are asking about vegetarian diets? Which foods will the nurse point out are not eaten when following the vegan diet?

Dairy products, meat, poultry, fish, and eggs

A nurse is reviewing the medical records of several male clients who are undergoing fertility assessments. The nurse is evaluating the clients' responses to the type of work that they do. The nurse determines that a client involved in which type of work is as risk for a lowered sperm count? Select all that apply.

Desk job Salesman who travels daily Long-distance truck driver

An 8-year-old boy's foster mother is concerned about three recent cavities found in his permanent teeth and reports the child eats a nutritional diet, doesn't eat junk food, and the town water supply is fluoridated. Which suggestion should the nurse prioritize to this mother in regard to the child's dental health?

Ensure that the child brushes his teeth after each meal and snacks.

What important information should the nurse give a client about the use of a diaphragm during menstruation?

Toxic shock syndrome is possible

A nursing instructor is discussing the various events that occur during a pregnancy. The instructor determines the session is successful after the students put the following events in the correct chronological order. Use all the options.

Union of an ovum and sperm Implantation of the zygote Formation of the placenta Heart begins beating Age of viability Fetus kicks actively

A client calls the nurse in a panic after a home pregnancy test indicates she is pregnant. She reports that that she consumed a lot of alcohol on the night that she thinks the pregnancy occurred. The next day she had taken several acetaminophen. For the past three weeks, she has had her usual nightly glass of wine with dinner but no other alcohol. What is an appropriate response for the nurse to make when the client questions if she has caused irreversible damage to the fetus?

"The fetus isn't exposed to the mother's blood until after it implants about six days after fertilization, so the first night is not an issue. But it's best to avoid alcohol while you're pregnant."

A nursing instructor is teaching nursing students about the various causes of intrauterine growth restriction. The instructor determines the session is successful after the students correctly choose which as recognized causative factors? Select all that apply.

Chromosomal abnormalities Fetal infection Pre-eclampsia Maternal illegal drug use

A nurse is caring for a pregnant client who has been diagnosed with lordosis. The nurse offers preventive measures for which consequence of lordosis when caring for this client?

Chronic backache

During pregnancy a woman has many psychological adaptations that must be made. The nurse must remember that the baby's father is also experiencing the pregnancy and has adaptations that must be made. Some fathers actually have symptoms of the pregnancy along with the mothers. What is this called?

Couvade syndrome

A pregnant client is diagnosed with hydramnios. The nurse explains that further testing will be done to determine if which conditions are present? Select all that apply.

Esophageal atresia Anencephaly

The nurse is discussing permanent birth control with a female patient and her husband. Which of the following is considered a permanent nonsurgical sterilization methods?

Essure

A client is desiring to start birth control and is questioning the nurse concerning the best options. Which option should the nurse point out is the most reliable?

Etonogestrel implant

The nurse is assessing a teenage client and notes his lower front teeth are slightly crossed over. The nurse points out to his caregiver that he should see an orthodontist about this to prevent which potential situation?

Even slight malocclusions make chewing and jaw function less efficient.

The 11-year-old arrives in the emergency department presenting with the following: nosebleed with no apparent trauma, disoriented, confused, difficulty walking, nausea, and coughing. The nurse should question the child concerning which potential activity?

Experimenting with an inhalant

A nurse observes a parent silently brushing her 8-year-old daughter's hair while the daughter is receiving chemotherapy. The child's eyes are closed and the parent tells the nurse, "This is something I have always done to relax her." What level of communication are the parent and child displaying?

Fifth Level: Peak Communication

The nurse is counseling the parents of a 10-year-old child who was caught stealing at school. Which topic should the nurse cover?

Finding out what is currently going on at home

A student nurse walks into a patient's room and states, "I am a student nurse who is going to take care of you today." Which level of communication is the student using?

First level

A nurse is passing a patient in the hallway and states, "Hope you are feeling better!" What level of conversation is this nurse having with this patient?

First level: Cliche Conversation

A clinic nurse is interviewing a young client during a subfertility work up. When the client asks the nurse what causes infertility, the nurse informs the client that the problem can rest with the man, the woman, or both. What does the nurse tell the client are common problem areas related to the woman? Select all that apply.

Ovulation Tubal transport Impaired implantation

A nurse is preparing a presentation for a health fair on the topic of vasectomy. Which information should the nurse point out in the information?

Relatively easy procedure with few complications

A woman in the third trimester of her first pregnancy expresses fear about the birth canal being wide enough for her to push the baby through it during labor. She is a petite person, and the baby seems so large. She asks the nurse how this will be possible. To help alleviate the patient's fears, the nurse should mention the role of the hormone that softens the cervix and collagen in the joints, which allows dilation and enlargement of the birth canal. This hormone is which of the following?

Relaxin

The mother of a 15-year-old boy confides in the nurse that she is concerned because her son is about to turn 16 and is pressuring her and her husband to buy him a motorcycle. Her husband is okay with the idea, but she is concerned about his safety. What information should the nurse mention to the mother regarding motorcycle safety? Select all that apply.

Require the son to wear a helmet. Require the son to wear long pants. Require the son to wear full body covering. Require the son to learn all relevant safety rules.

A nurse caring for a young patient with sickle-cell crisis always establishes eye contact, uses a gentle tone of voice, listens carefully to whatever the patient says, and uses touch appropriately. Which of the following is the nurse demonstrating by these actions?

Warmth


Kaugnay na mga set ng pag-aaral

Unit 1: Limits and Their Properties

View Set

Corporate Finance Test #3 (Ch. 20, 21, 23, 24, 26, 27)

View Set

Biology Chapter 48 Neurons, Synapses, and Signaling

View Set

Chapter 18 and Smartbook - Principles of Finance

View Set

CM 333: Quiz 2 (Ch. 3) Accident Prevention Program

View Set